Un operador "hermitiano" con valores propios imaginarios

Dejar

H = X ^ 3 pags ^ + pags ^ X ^ 3
dónde pags ^ = i d / d X . Claramente H = H , porque H = T + T , dónde T = X ^ 3 pags ^ . En este sentido H es "formalmente" autoadjunto.

Resulta que

ψ λ ( X ) = 1 | X | 3 / 2 mi λ / 4 X 2 L 2 [ , ]
es una función propia de H . En realidad

H ψ λ ( X ) = i λ ψ λ ( X ) .

Si pasamos por la prueba habitual de que los operadores hermitianos tienen valores propios reales, vemos que algo tiene que salir mal en los siguientes pasos:

ψ λ ( X ) | H ψ λ ( X ) = H ψ λ ( X ) | ψ λ ( X ) = H ψ λ ( X ) | ψ λ ( X ) = ψ λ ( X ) | H ψ λ ( X ) .

Claramente H ψ λ ( X ) = i λ ψ λ ( X ) L 2 . Así que todas estas operaciones parecen estar bien definidas.

Nuestro libro sugiere que miremos T ψ λ ( X ) . Resulta que T ψ λ ( X ) , T ψ λ ( X ) L 2 . Así que cuando estamos escribiendo

ψ λ ( X ) | H ψ λ ( X )

realmente estamos escribiendo

ψ λ ( X ) | ( T + T ) ψ λ ( X ) = ψ λ ( X ) | T ψ λ ( X ) + ψ λ ( X ) | T ψ λ ( X )

donde estos productos internos ya no están definidos. ¿El hecho de que estos dos productos internos no estén definidos conduce a que este operador aparentemente (y "formalmente") autoadjunto tenga valores propios imaginarios? ¿Y cómo?

¡Pulcro! Esta es realmente una pregunta puramente matemática, por lo que alguien podría querer moverla a math.SE, pero me gusta. :)
¿Puede mostrar explícitamente que su ψ es de hecho un valor propio de su hamiltoniano?
@BMS Me gusta, ¿quieres que lo escriba? Es un cálculo rápido. Solo hay que tener cuidado al calcular d / d X | X | 3 / 2 = ( 3 / 2 ) s gramo norte ( X ) | X | 5 / 2 .
Este problema es el Ejemplo 3 en F. Gieres, Sorpresas matemáticas y el formalismo de Dirac en mecánica cuántica, arXiv:quant-ph/9907069 , véase la pág. 6, 39, 45-47.
@Qmechanic Esa es una lista interesante de preguntas que vale la pena leer
Siguiendo Mathematica verifica que el valor propio es correcto con el signo -: Block[{H = x^3 1/ID[#, x] + 1/ID[x^3 #, x] &, [Psi] = 1/Abs[ x]^(3/2) Exp[- [Lambda]/(4 x^2)]}, FullSimplify[H@[Psi] == -I [Lambda] [Psi], Suposiciones :> x [Elemento] Reales ]]
@BMS: Empezar desde ψ := | X | 3 2 Exp [ λ 4 X 2 ] C ( R ) L 2 ( R ) , dónde λ > 0 es una constante positiva. Rendimientos de diferenciación X ψ = ( λ 2 X 3 3 2 X ) ψ . Con operador de orden normal i H = X 3 X + X X 3 = 2 X 3 X + 3 X 2 en la representación de Schroedinger, esto conduce a un valor propio imaginario i H ψ = λ ψ .

Respuestas (3)

El problema con este hamiltoniano es que existe una diferencia entre los operadores simétricos/hermitianos y los operadores autoadjuntos. Parece que un matemático quisquilloso está metiendo agujeros en todo, pero de hecho es importante:

En general, los dominios de A ^ y A ^ no coincidan Si A ^ = A ^ en D ( A ^ ) , después D ( A ^ ) D ( A ^ ) sostiene y A ^ se llama simétrica o hermitiana. Si, además, D ( A ^ ) = D ( A ^ ) , después A ^ se llama autoadjunto.

Los teoremas importantes de existencia y realidad para valores propios y vectores propios suelen ser solo para operadores autoadjuntos. Esto se aclara en la página 13 de su libro de texto . Si bien su operador es simétrico, es poco probable que sea autoadjunto.

Más específicamente, H está densamente definido y por lo tanto tiene un adjunto H , que es un operador en algún dominio D ( H ) que satisface ϕ | H ψ = H ϕ | ψ para todos ψ D ( H ) y ϕ D ( H ) . Tu verdadero trabajo es caracterizar los dominios de ambos operadores y ver si coinciden, o averiguar si H se puede extender a un dominio más grande de modo que el dominio del adjunto coincida con el dominio original. Nada de eso es particularmente fácil.


Sin embargo, la cuestión es que estos problemas matemáticos muy rara vez surgen por sí solos y suelen ir acompañados de problemas en el problema clásico correspondiente. Esto está hermosamente aclarado, junto con una clara exposición de los hechos matemáticos necesarios, en el artículo.

Síntomas clásicos de las enfermedades cuánticas. Chengjun Zhu y John R. Klauder. Soy. J. física. 61 núm. 7, 605 (1993) .

El punto principal es que, a menos que el problema clásico tenga soluciones bien definidas para todos los tiempos y para todas las condiciones iniciales, realmente no tiene por qué quejarse del comportamiento inesperado en la contraparte cuántica .

Para su modelo, el hamiltoniano clásico H = 1 2 ( q 3 pags + pags q 3 ) = q 3 pags produce las ecuaciones de Hamilton

{ pags ˙ = H q = 3 q 2 pags , q ˙ = H pags = q 3 .
Estos son bastante fáciles de resolver, y las soluciones no se comportan bien:
{ 1 2 q 2 = t 0 t , pags 0 2 pags 2 = ( t 0 t ) 3 ,
dónde t 0 y pags 0 son constantes de integración. Tenga en cuenta, en particular, que no hay soluciones (reales) después de un cierto tiempo t 0 = t en + 1 2 q ( t en ) 2 . Entonces, ¿cómo esperas una física razonable de la versión cuantificada de esto?


Finalmente, como coda, permítanme abordar el comentario muy interesante de Qmechanic. Es cierto que para un sistema físico dado, tendrás un solo hamiltoniano y muchos otros observables físicos. Entonces, ¿cómo se puede dar sentido a esta construcción para un observable arbitrario? Contestaría que cualquier observable arbitrario puede considerarse hamiltoniano, y que cosas como el espectro se derivan de propiedades como la evolución temporal correspondiente.

Tome un observable físico arbitrario autoadjunto, tan bueno como sea necesario. A ^ en un sistema físico con espacio de estado H . Incluso si no tiene sentido físico, definitivamente puede postular el flujo asociado con ese observable, es decir, la curva t | ψ ( t ) en H que obedece a la ecuación de Schrödinger

i t | ψ ( t ) = A ^ | ψ ( t ) .
Si desea poner en juego el espectro, puede resolver esta ecuación descomponiendo el flujo en la base propia del observable, por lo que | ψ ( t ) = norte ψ norte mi i a norte t | norte , dónde A ^ | norte = a norte | norte y ψ norte = norte | ψ ( 0 ) . (Aquí hay que suponer que | ψ ( 0 ) es lo suficientemente "general" o "aleatorio" como para que todos los ψ norte son distintos de cero.

Sin embargo, la pregunta es ¿cómo se puede extraer el espectro de la evolución temporal? La respuesta a eso es la transformada de Fourier en el dominio de la frecuencia: definir

| ψ ~ ( ω ) = d t mi i ω t | ψ ( t )
y vea lo que le hace la descomposición del vector propio:
| ψ ~ ( ω ) = d t mi i ω t norte ψ norte mi i a norte t | norte = norte ψ norte | norte d t mi i ( ω a norte ) t = norte d ( ω a norte ) ψ norte | norte .
Es decir: la transformada de Fourier de la A ^ -el flujo inducido es igual a un número de picos en los valores propios de A ^ , y los vectores propios son los coeficientes.

Para decirlo de otra manera, esto ofrece un medio para obtener el espectro a partir de la evolución temporal: resolverlo de alguna manera, transformar la solución de Fourier y luego leer los valores propios del soporte de la transformada y los vectores propios del valor en esos puntos. . De hecho, esta es una técnica útil y tiene un despliegue bastante amplio en soluciones numéricas de ciertas clases de problemas. (Para obtener más información, consulte el excelente libro de texto de David Tannor Introducción a la mecánica cuántica: una perspectiva dependiente del tiempo ( Ebookee )).

... y, por supuesto, si intentara diagonalizar de esta manera a un operador con el tipo de problemas descritos anteriormente, se dirigiría directamente a los problemas. ¡Seguramente no es razonable pedirle al flujo cuántico que se comporte bien cuando el flujo clásico no lo hace!

Solo un pequeño comentario del abogado del diablo: el operador hamiltoniano (que describe la evolución temporal del sistema) podría en principio ser diferente del operador de OP H (que entonces es solo un observable particular del sistema).
@Qmechanic Vea la respuesta en la publicación.
@EmilioPisanty, respuesta interesante, especialmente el segundo artículo. Me preguntaba si podría explicar exactamente cómo los dominios D ( H ) y D ( H ) diferir de. Después de todo, parece (prima facie) D ( H ) = D ( T ) D ( T ) = D ( H ) .
@EmilioPisanty además, ¿cuál es el paso erróneo en el siguiente cálculo? ψ λ ( X ) | H ψ λ ( X ) = H ψ λ ( X ) | ψ λ ( X ) = H ψ λ ( X ) | ψ λ ( X ) = ψ λ ( X ) | H ψ λ ( X ) . Todos los productos internos se evalúan presumiblemente en el mismo espacio de Hilbert.
+1 muy bueno! ¡Casi me hace desear haber hecho un análisis funcional! (Lamentablemente, nuestro departamento de matemáticas se redujo a un plan de estudios básico de "matemáticas para ingenieros"; algunos de nosotros logramos pasar antes de la mayor parte del destripamiento, pero eso aún nos dejó ciertos vacíos: S)

Aunque la respuesta de Emilio es perspicaz, no creo que responda directamente a su pregunta. Intentaré hacer eso aquí. Esta respuesta se desarrolla en dos partes:

  1. Mostraremos que el operador que trata de escribir es hermético con el dominio apropiado, pero que no es autoadjunto y no tiene extensiones autoadjuntas.

  2. Mostraremos que sus manipulaciones formales tienen errores.

Parte 1.

Establecimos = 1 para mayor comodidad en todo momento, y dejar S ( R ) denote el espacio de Schwartz. Recuerda que el L 2 ( R ) el producto interior se define como sigue:

ψ , ϕ = d X ψ ( X ) ϕ ( X )

Vamos a utilizar la siguiente definición que aparece en la página 138 de Methods of Modern Mathematical Physics Volumen II de Reed y Simon (análisis de Fourier, autoadjunción):

Definición. Para un operador simétrico A , definimos sus índices de deficiencia por

norte + ( A ) = oscuro k mi r ( i yo A ) norte ( A ) = oscuro k mi r ( i yo + A )

También vamos a necesitar el siguiente resultado que es parte de un corolario en la página 141 de Reed y Simon:

Lema. Dejar A ser un operador hermitiano cerrado con índices de deficiencia norte + ( A ) y norte ( A ) , después A es autoadjunto si y solo si norte + ( A ) = 0 = norte ( A ) , y A tiene al menos una extensión autoadjunta si y solo de norte + ( A ) = norte ( A ) .

Con este lema, podemos probar la siguiente afirmación. Lo que vamos a probar aquí nos dice que no hay manera de definir H operador autoadjunto en algún dominio en L 2 ( R ) .

Reclamar. El operador H con dominio D ( A ) = S ( R ) definido por

H ψ ( X ) = i X 3 d ψ d X ( X ) i d d X ( X 3 ψ ( X ) )
es cerrado y hermitiano pero no auto-adjunto. Es más, H no tiene extensiones autoadjuntas en L 2 ( R ) .

Prueba. mostraremos que A es hermético y cerrado pero eso norte ( H ) = 1 tiempo norte + ( H ) = 0 . El resultado deseado se sigue inmediatamente del lema. Para mostrar que H es hermitiano, basta mostrar que ψ , H ϕ = H ψ , ϕ para todos ϕ , ψ D ( H ) = S ( R ) . Tenemos

ψ , H ϕ = d X ψ ( i X 3 d ϕ d X i d d X ( X 3 ϕ ) ) = 2 i ψ X 3 ϕ | + i d X ( d d X ( ψ X 3 ) + d ψ d X ( X ) X 3 ) ϕ = d X ( i d d X ( ψ ( X ) X 3 ) i d ψ d X X 3 ) ϕ = H ψ , ϕ .
El término de frontera en la segunda igualdad desapareció porque ϕ está disminuyendo rápidamente. Este operador está cerrado (es cierto que esto es algo que no he podido probar). Porque H es hermítico, su adjunto H tiene la misma acción sobre los elementos de su dominio que H sí mismo. Además, deja D el conjunto de todos ψ L 2 ( R ) para cual H ψ está bien definido y también es un elemento de L 2 ( R ) . Luego, el cálculo realizado anteriormente para demostrar la hermiticidad muestra que si ϕ D , después ϕ , H ψ = H ϕ , ψ para todos ψ D ( H ) , asi que D ( H ) = D . En particular, este dominio es mayor que el de H que por lo tanto no es auto-adjunto.

Ahora si ψ k mi r ( i yo A ) , después ψ obedece a la siguiente ecuación diferencial:

i ψ ( i X 3 d ψ d X i d d X ( X 3 ψ ) ) = 0
Esta ecuación diferencial se puede simplificar para dar
( 1 + 3 X 2 ) ψ + 2 X 3 d ψ d X = 0
por X > 0 y X < 0 , podemos separar variables e integrar para resolver esta ecuación diferencial. El resultado es (utilicé mathematica para esto)
ψ > ( X ) = mi 1 / ( 4 X 2 ) X 3 / 2 + C > ψ < ( X ) = mi 1 / ( 4 X 2 ) ( X ) 3 / 2 + C <
Estas soluciones divergen en el origen, por lo que nuestra ecuación diferencial no produce una L 2 ( R ) solución. Esto da k mi r ( i yo A ) = { 0 } asi que norte + ( H ) = 0 . Por otro lado, si ψ k mi r ( i yo + A ) después
i ψ + ( i X 3 d ψ d X i d d X ( X 3 ψ ) ) = 0
Esta ecuación diferencial se puede simplificar para dar
( 1 3 X 2 ) ψ 2 X 3 d ψ d X = 0
que admite la solución normalizada
ψ ( X ) = { 1 2 1 | X | 3 / 2 mi 1 / ( 4 X 2 ) , X 0 0 , X = 0
De hecho, esta es (hasta la normalización) exactamente la función que escribió en la declaración de la pregunta original. Esta función está en D ( H ) . Resulta que k mi r ( i yo + A ) = s pags a norte { ψ 1 } de modo que norte ( H ) = 1 , como se desee .

Parte 2.

En cuanto a sus manipulaciones, incluso si fuera a ampliar el dominio de H para incluir ψ λ , estarían equivocados. Observe, por ejemplo, que obtuvo un término límite distinto de cero en el siguiente cálculo:

ψ λ , H ψ λ = d X ψ λ ( i X 3 d ψ λ d X i d d X ( X 3 ψ λ ) ) = 2 i ψ λ X 3 ψ λ | + i d X ( d d X ( ψ λ X 3 ) + d ψ λ d X ( X ) X 3 ) ψ λ = 2 i s gramo norte ( X ) mi λ / ( 2 X 2 ) | + i d X ( d d X ( ψ λ X 3 ) + d ψ λ d X ( X ) X 3 ) ψ λ = 4 i + d X ( i d d X ( ψ ( X ) X 3 ) i d ψ d X X 3 ) ψ λ = 4 i + H ψ λ , ψ λ
que podríamos haber visto más fácilmente simplemente observando que
ψ λ , ψ λ = 2 λ
lo que da
ψ λ , H ψ λ = i λ ψ λ , ψ λ = 2 i H ψ λ , ψ λ = ( i λ ) ψ λ , ψ λ = 2 i
En particular, ambos cálculos muestran que
ψ λ , H ψ λ H ψ λ , ψ λ
en contradicción con lo que afirmas.

Solo un comentario sobre tu buena respuesta. En realidad para aplicar la tecnología de índices de carencia no es necesario exigir que H es cerrado, simétrico (no sólo hermitiano) es suficiente. Simétrico significa hermitiano con dominio denso. Si H es simétrico, A es una extensión autoadjunta de H si y solo si es una extensión autoadjunta del cierre H ¯ que es cerrado y simétrico. Es más norte ± ( H ) = norte ± ( H ¯ ) , por lo que tratar con H mismo como tú lo hiciste (y H no está cerrado) es completamente suficiente.
@V.Moretti Muchas gracias por esa aclaración; eso me hace sentir mejor acerca de la respuesta. Dado que yo, y asumo que otros, seguramente no están tan familiarizados con el análisis funcional como usted, ¿me estaría señalando una o dos referencias que podrían discutir la observación que ha hecho con algún detalle en caso de que conozca alguna?
Hola Josh. Desafortunadamente, parece que la literatura se divide en dos conjuntos. Un conjunto de libros de texto como Reed-Simon, Rudin, Prugovecki, etc. asume que el operador debe ser simétrico y cerrado, otro conjunto como Dunford-Schwartz, Teschl, etc. asume solo simetría. Nunca encontré un libro que discutiera la interacción en detalle, así que produje mis propias pruebas (en realidad muy elementales) y las estoy insertando en la segunda edición de mi libro. Por favor envíeme un correo electrónico a mi dirección institucional (moretti@science.unitn.it) Mañana enviaré copia de las pruebas.
@V.Moretti Ah, ya veo, eso es desafortunado. Te enviaré un correo; ¡gracias!

Este problema está planteado y resuelto por: François Gieres su artículo: Sorpresas matemáticas y el formalismo de Dirac en la mecánica cuántica .

Se da en el ejemplo 3 página 6 y su solución se da en: (3a) página 39 y continúa en las páginas 45 y 46 donde se formula y aplica al problema el criterio de von Neumann para la autoadjunción.

Como se explica en el artículo, la razón básica por la que la función ψ λ no es una función propia de H es que no pertenece al espacio de Schwartz S ( R ) (de funciones rápidamente decrecientes) que el dominio mutuo de definición de ambos X ^ y pags ^ .

En la segunda parte se demuestra que ψ λ es una función propia de H y no hay manera de extender el dominio de H para hacerlo auto adjunto.